GENERAL PRACTITIONER EXAM
Question Summary
0 of 100 questions completed
Questions:
- 1
- 2
- 3
- 4
- 5
- 6
- 7
- 8
- 9
- 10
- 11
- 12
- 13
- 14
- 15
- 16
- 17
- 18
- 19
- 20
- 21
- 22
- 23
- 24
- 25
- 26
- 27
- 28
- 29
- 30
- 31
- 32
- 33
- 34
- 35
- 36
- 37
- 38
- 39
- 40
- 41
- 42
- 43
- 44
- 45
- 46
- 47
- 48
- 49
- 50
- 51
- 52
- 53
- 54
- 55
- 56
- 57
- 58
- 59
- 60
- 61
- 62
- 63
- 64
- 65
- 66
- 67
- 68
- 69
- 70
- 71
- 72
- 73
- 74
- 75
- 76
- 77
- 78
- 79
- 80
- 81
- 82
- 83
- 84
- 85
- 86
- 87
- 88
- 89
- 90
- 91
- 92
- 93
- 94
- 95
- 96
- 97
- 98
- 99
- 100
Information
Hi, Welcome to General Practioner Exam
You have already completed the Exam before. Hence you can not start it again.
Exam is loading...
You must sign in or sign up to start the Exam.
You have to finish following Exam, to start this Exam:
Results
0 of 100 questions answered correctly
Your time:
Time has elapsed
You have reached 0 of 0 points, (0)
Average score |
|
Your score |
|
Categories
- Not categorized 0%
Pos. | Name | Entered on | Points | Result |
---|---|---|---|---|
Table is loading | ||||
No data available | ||||
- 1
- 2
- 3
- 4
- 5
- 6
- 7
- 8
- 9
- 10
- 11
- 12
- 13
- 14
- 15
- 16
- 17
- 18
- 19
- 20
- 21
- 22
- 23
- 24
- 25
- 26
- 27
- 28
- 29
- 30
- 31
- 32
- 33
- 34
- 35
- 36
- 37
- 38
- 39
- 40
- 41
- 42
- 43
- 44
- 45
- 46
- 47
- 48
- 49
- 50
- 51
- 52
- 53
- 54
- 55
- 56
- 57
- 58
- 59
- 60
- 61
- 62
- 63
- 64
- 65
- 66
- 67
- 68
- 69
- 70
- 71
- 72
- 73
- 74
- 75
- 76
- 77
- 78
- 79
- 80
- 81
- 82
- 83
- 84
- 85
- 86
- 87
- 88
- 89
- 90
- 91
- 92
- 93
- 94
- 95
- 96
- 97
- 98
- 99
- 100
- Unanswered
- Answered
- Review
-
Question 1 of 100
1. Question
1 pointsA 14 year old boy with long history of chronic abdominal discomfort and diarrhea has hemarthrosis of the right knee joint and a recent history of protracted bleeding from cuts or scrapes. Family history of bleeding disorders is not present. Exam shows a patient at the 5th percentile for both height and weight; an actively bleeding rectal fissure is also noted. Both PT and PTT are prolonged. Investigation of the blood is likely to reveal low levels of which of the following?
Correct
Incorrect
Explanation:
Low levels of factors II, VII, IX, and X are seen in vitamin K deficiency, leading to prolonged prothrombin time (PT) and partial thromboplastin time (PTT). Vitamin K deficiency is occasionally severe enough in obstructive jaundice, pancreatic disease, or small bowel disease to cause a bleeding diathesis. This patient has evidence of small bowel disease and a history suggestive of Crohn disease (chronic abdominal discomfort, diarrhea and fever). Crohn disease, which is also characterized by rectal fissures, growth retardation, and malabsorption, causes malabsorption of fat soluble vitamins (A, D, E, and K) by several mechanisms. It most often involves the terminal ileum, which is responsible for the recycling of bile acids necessary for the transport and proper absorption of lipids. Small intestinal Crohn disease itself can cause malabsorption by reducing the surface area available for absorption of nutrients. Finally, the disease can cause the development of fistulae, which can lead to exclusion of loops of bowel, also reducing available absorptive surface area. Factor VIII deficiency is the cause of hemophilia A. This answer is incorrect because hemophilia A is characterized by an elevated PTT but a normal PT, since only factor VIII is involved. Although hemophilia A can cause gastrointestinal hemorrhage and pain, a 6 month crisis with abdominal discomfort as the only symptom would be extremely rare. Also, hemophilia would likely be characterized by black tarry stools rather than diarrhea. Hemophilia A is inherited as an X linked recessive; thus, affected individuals are usually male, whereas carriers are female. Low levels of factor IX is the cause of Christmas disease. Like hemophilia A, factor IX deficiency is characterized by prolonged APTT and normal PT. Specific coagulation factor assays distinguish these two diseases, as they are otherwise identical in both presentation and inheritance. Low levels of factors II, V, VII, IX, and X could be characteristic of liver disease, but in such a case both PT and APTT would be elevated. Note that in liver disease, all other factors (except for Factor VIII) would also be low. Low levels of von Willebrand´s factor cause a prolonged or normal APTT, a normal PT, and a prolonged bleeding time. Von Willebrand disease is inherited in an autosomal dominant pattern with incomplete penetrance.
-
Question 2 of 100
2. Question
1 pointsA 65 year old man has vague complaints of fatigue. Exam shows splenomegaly without lymphadenopathy. His hematocrit is 30%, platelet count is 80,000/mm3, and his leukocyte count is 2500/mm3. Bone marrow biopsy shows “fried egg” cells expressing pan B cell markers CD19 and CD20. The most likely diagnosis is
Correct
Incorrect
Explanation:
Hairy cell leukemia is characterized by pancytopenia, often with massive splenomegaly. Lymphadenopathy is unusual. The proliferating cells express the pan B-cell markers CD 19 and CD20 as well as monocyte markers. Histologically, the cells display fine, hairlike projections. A “fried-egg” appearance of bone marrow biopsies arises because of the fixation artifacts from the hair like projections on many of the cells. Tartrate resistant acid phosphatase (not mentioned in this question) is virtually diagnostic of hairy cell leukemia. Interferon-alfa is used for treatment. Aplastic anemia, or marrow aplasia, is also characterized by pancytopenia; however, the marrow would be hypocellular, with fatty replacement of hematopoietic cells. Hereditary spherocytosis is due to a defect in the membrane of the red cells and would be accompanied by abnormal red cell morphology. Splenomegaly is typical, but other cell lines (white blood cells, platelets) are not affected. Remember that spherocytes exhibit increased osmotic fragility. Hodgkin disease is a lymphoma characterized by the presence of Reed-Sternberg cells in a variable lymphocytic infiltrate. Sezary syndrome is related to mycosis fungoides, a cutaneous T cell (not B-cell) lymphoma. In both disorders, there is a characteristic “cerebriform” cell, which circulates in the blood in Sezary syndrome but remains confined to the skin in most cases of mycosis fungoides. -
Question 3 of 100
3. Question
1 pointsUltrasound at 18 weeks gestation in a diabetic female reveals a male fetus with an endocardial cushion defect. Other abnormalities include increased nuchal thickening and a double bubble sign. The conditions that is likely to have contributed to this set of findings is which one of the following?
Correct
Incorrect
Explanation:
Diabetic mothers are likely to have children with congenital abnormalities depending on pre-conception and first trimester blood sugar control. 40% of Down syndrome babies have atrioventricular septal defects, as this fetus. The double bubble sign suggest duodenal atresia, which again suggests Down syndrome.
GI malformations occur in 6% of Down´s patients-most commonly duodenal atresia and Hirschsprung´s disease. -
Question 4 of 100
4. Question
1 pointsLeast true statement regarding IgA nephropathy is which one of the following?
Correct
Incorrect
Explanation:
IgA nephropathy (Berger´s disease) is the most common glomerulonephritis worldwide, and characteristically affects young males, presenting with frank haematuria after an episode of pharyngitis. However it may also present with proteinuria, microscopic haematuria, renal failure or hypertension. It is probably part of a Spectrum of disease with Henoch Schoenlein Purpura, which presents with arthritis, rash, abdominal pain and nephritis. In both there are mesangial IgA deposits in the kidney.
-
Question 5 of 100
5. Question
1 pointsWhich of the following statements concerning abnormalities of the haemoglobin molecule is true?
Correct
Incorrect
Explanation:
Alpha thalassaemia is due to abnormalities of the alpha chain. Persistence of HbF has survival advantages in severely affected A subjects.
C-alpha 16, beta 11 e-Hb electrophoresis in the adult rather than oligonucleotide probes as used in the fetus. -
Question 6 of 100
6. Question
1 pointsA 24-year-old German male is diagnosed with diabetes. He has frontal balding, and tells you that he has previously been diagnosed with a cardiomyopathy. Which of the following genetic phenomena is associated with this genetic condition?
Correct
Incorrect
Explanation:
The suggestion here is that the patient has myotonic dystrophy. Myotonic dystrophy is autosomal dominant and exhibits anticipation as the number of trinucleotide repeats undergo triplet expansion, resulting in the disease presenting at a younger age, or with greater disease severity along the generations.
Examples of chromosome instability syndromes are Fanconi anaemia, ataxia telangiectasia and Bloom syndrome. -
Question 7 of 100
7. Question
1 pointsA 19-year-old girl underwent emergency splenectomy after a domestic accident. Which one of the following organism is most likely to cause life-threatening infection in the future?
Correct
Incorrect
Explanation:
Following splenectomy a person is at risk of Streptococcus pneumoniae, Haemophilus influenzae, Nesseria meningitidis, Escherichia coli and Pseudomonas aeruginosa.
By far the most common is Streptococcus pneumoniae, which can cause life-threatening infection. -
Question 8 of 100
8. Question
1 pointsA 19-year-old attending the A+E department is noted to have central cyanosis. She is perfectly well but was told to go to A+E by her friends who said she looked blue. What is the most likely cause?
Correct
Incorrect
Explanation:
This is typical of methemoglobinemia which may be caused by nitrates. Polycythemia Vera is less likely as it will have more symptoms. Cold and Raynaud´s are causes of peripheral cyanosis. Carbon monoxide is less likely to present like this.
-
Question 9 of 100
9. Question
1 pointsWhat tumor marker is most likely to be elevated in pheochromocytoma?
Correct
Incorrect
Explanation:
Urine metanephrine testing is primarily used to help detect and rule out pheochromocytomas in symptomatic patients. It may also be ordered to help monitor the effectiveness of treatment when a pheochromocytoma is discovered and removed and to monitor for recurrence. The alpha-fetoprotein (AFP) tumor marker may be elevated in HCC. Prostate specific antigen (PSA) may rise in prostate carcinoma. CA- 125 can be elevated in ovarian carcinoma. Carcinoembryonic antigen (CEA) may be elevated in colon carcinoma.
-
Question 10 of 100
10. Question
1 pointsAll of the following statements are true regarding uncontrolled diabetes mellitus EXCEPT
Correct
Incorrect
Explanation:
Patients with uncontrolled diabetes are not at higher risk for gastric cancer than the general population. All of the other statements are true.
-
Question 11 of 100
11. Question
1 pointsAll of the following statements regarding thrombotic thrombocytopenic purpura are true EXCEPT
Correct
Incorrect
Explanation:
Emergency splenectomy used to be a common form of therapy for TTR but has been supplanted by other modalities, such as exchange transfusion. All of the other statements are true.
-
Question 12 of 100
12. Question
1 pointsA tall male with gynecomastia and testicular atrophy has a testicular biopsy that shows sparse, completely hyalinized seminiferous tubules with a complete absence of germ cells and only rare sertoli cells. Leydig cells are present in large clumps between the hyalinized tubules. The genetic disorder suspected in this patient is
Correct
Incorrect
Explanation:
The testicular changes described are those observed in Klinefelter syndrome, most often due to 47,XXY (genetics. One in 11000 male births have this abnormality, and there is an increased incidence with increased maternal age; in 60% of cases, the extra X is of maternal origin. The males are typically infertile with small testes, a small penis, and low testosterone levels. Affected individuals have a eunuchoid body shape with long extremities. Almost 50% of these patients have eunuchoid. Other, less common findings include diabetes mellitus, scoliosis, and mild mental retardation.
Testicular feminization syndrome (choice A) is due to a genetically determined unresponsiveness to testosterone caused by abnormalities in androgen receptors, producing a phenotypic female in an individual with 46, XY chromosomes.
Trisomy 18 (choice B) is Edwards syndrome, characterized by mental retardation, micrognathia, delicate facial features low-set ears, cardiac defects, renal defects, and rocker- bottom feet. Survival is rare beyond 1 year.
Trisomy 21 (choice C) or Down syndrome is the most common trisomy. Characteristics include oblique palpebral fissures, epicanthal folds endocardial cushion defects, simian creases, and high-arched palate, among other anomalies. Most patients have mental retardation.
Turner syndrome, 45.XO (choice D) produces a sterile but phenotypic female individual of short stature with webbing of the neck. -
Question 13 of 100
13. Question
1 pointsAdult respiratory distress syndrome (ARDS) is NOT associated with
Correct
Incorrect
Explanation:
ARDS is associated with any disorder that causes severe lung injury. All of these conditions except can damage the lungs. No direct association has been demonstrated between ulcerative colitis and ARDS.
-
Question 14 of 100
14. Question
1 pointsLung biopsy of a 28 year old with shortness of breath reveals a necrotizing vasculitis that is accompanied by granuloma formation. The vasculitis consists of a disproportionate number of eosinophils. She is hepatitis B surface antigen negative, and shows no renal abnormalities. The most likely diagnosis is
Correct
Incorrect
Explanation:
Churg Strauss syndrome is consistent with all of the findings: dyspnea, necrotizing vasculitis with granuloma formation, eosinophilia, hepatitis B surface antigen negativity, and no renal abnormalities. Polyarteritis nodosa demonstrates no pulmonary involvement, while Wegener´s granulomatosis and Goodpasture´s syndrome also involve the kidneys. Extrinsic asthma will not present with a vasculitis.
-
Question 15 of 100
15. Question
1 pointsFALSE statement regarding vomiting is which one of the following?
Correct
Incorrect
Explanation:
omiting secondary to gastric outlet obstruction or impaired motility usually is delayed and occurs more than an hour postprandially. Vomiting is the forceful expulsion of gastric contents and is under two brain control centers. The vomiting center is excited by visceral afferent fibers from the gastrointestinal tract. This center serves to coordinate other medullary centers to produce a patterned response to noxious stimuli and disease processes. An intact vomiting center is necessary for the chemoreceptor trigger zone to cause vomiting. The presence of bile in vomitus excludes obstruction at the gastric pylorus.
-
Question 16 of 100
16. Question
1 pointsA person presents with headache, fever, and menigismus associated with a CSF pleoscytosis. Gram stain and cultures are negative and viral meningitis is diagnosed. True statement regarding viral meningitis is
Correct
Incorrect
Explanation:
Viral meningitis is usually benign and self-limiting, unlike bacterial meningitis which can be life threatening. Contacts of patients with bacterial meningitis, not viral meningitis, need antibiotic therapy. Antibiotics do not play a role in the management of viral meningitis.
-
Question 17 of 100
17. Question
1 pointsFALSE statement regarding osmotic diarrhea is which one of the following?
Correct
Incorrect
Explanation:
Either fasting or cessation of ingestion of the offending agent prevents the diarrhea. Osmotic diarrhea results from the accumulation of nonabsorbed solutes in the intestinal lumen. It may result from the ingestion of poorly absorbable solutes, such as saline purgatives. Maldigestion of ingested food, such as that occurring in lactase deficiency may also cause it. Failure of a mucosal transport mechanism, such as occurs in glucose galactose malabsorption, may also cause it. These osmotically active agents result in the retention of water and salts within the intestinal lumen.
-
Question 18 of 100
18. Question
1 pointsCholecystectomy is performed on a South Asian immigrant who presents with severe right upper quadrant pain. Gall bladder shows a large number of darkly colored faceted stones with irregular shapes. Which infectious agent causes this type of stones?
Correct
Incorrect
Explanation:
The stones are pigment stones, composed of relatively pure calcium bilirubinate. These stones are less common than cholesterol stones. Risk factors for pigment stones include hepatic cirrhosis (unclear mechanism), hemolytic anemias, and liver fluke infection notably by the oriental liver fluke Clonorchis sinensis. Fasciolopsis buski is a large intestinal fluke that can cause abdominal pain, diarrhea, and intestinal mucosal abscesses. Fasciola, not Fasciolopsis, species are another form of liver fluke besides Clonorchis. Paragonimus westermani is a lung fluke that can cause cough, bronchiectasis, and hemoptysis. Schistosoma haematobium is a blood fluke that tends to infect pelvic veins and may cause bladder disease (including cancer). Taenia saginata is the beef intestinal tapeworm, and may cause intestinal obstruction.
-
Question 19 of 100
19. Question
1 pointsA research laboratory of microbiology is studying the progression of disease in mice infected with Neisseria meningitidis. After inoculation of a standard number of organisms, the development of petechial hemorrhage in the footpads is followed as a function of bacterial division (colony forming units per mL of blood) as shown below:
Time (minutes) CFU/Ml of blood Petechiae 0 1 negative 30 1 negative 60 2 negative 120 8 + 180 32 ++ 240 128 +++ 300 220 +++ In the bacterial growth curve, at what point does the release of petechia producing toxic factors reach observable levels?
Correct
Incorrect
Explanation:
The development of petechial hemorrhage during infection with a gram negative organism reflects the release of endotoxin at pathogenic levels. Endotoxin (lipopolysaccharide) is a constituent of the outer membrane of gram negative bacteria. Therefore, in most cases of gram negative infection, the cells have to begin to die and be degraded for the toxin to be released. Meningococcal infection, caused by Neisseria meningitidis, is the exception to this rule, since the pathogen is the only gram negative organism that overproduces outer membrane fragments throughout its life. Clinically, this means that patients with meningococcal meningitis present with very early signs of endotoxin mediated disease. In relation to the bacterial growth curve demonstrated in these data, Petechiae begin to be observed at the 120 minute time point. The number of bacteria present at 120 minutes is 8 times the number present at time zero, which reflects that the bacteria have undergone 3 generations of binary fission. Thus, Petechiae are first observed in the logarithmic or exponential phase of the growth curve. During the acceleration phase, cell numbers are beginning to increase, reflecting the beginning of successful binary fission in a culture. In this experiment, the acceleration phase occurred between the 30 minute and the 60 minute time point, and Petechiae were not observed at this time. In the decline phase, cell death exceeds cell proliferation, and cell numbers therefore decline. The data presented here do not include the decline phase of growth, since bacterial numbers are still increasing at the final time point. In addition, petechiae first appear well before bacterial multiplication rates begin to slow at the 300 minute time point. In meningococcal meningitis, endotoxin is released before cell death begins. During the lag phase of a growth curve, bacteria are not dividing. In the experiment shown, the lag phase occurs in the first 30 minutes of elapsed time. There are no Petechiae observed in the footpads at this time. During the retardation phase of a growth curve bacterial numbers increase but at a lower rate than in the logarithmic phase. This is due to the fact that nutrients become limiting and toxic products begin to build up in the system, so that some cells begin to die while others are still undergoing binary fission. In this experiment, the retardation phase has begun following the 240 minute time point, and Petechiae were first observed well before this time.
-
Question 20 of 100
20. Question
1 pointsA 49-year-old has Gardener´s syndrome. She now has a small palpable nodule in her neck. Thyroid function tests and thyroid antibodies are normal. Which of the following tumors is she at increased risk of developing?
Correct
Incorrect
Explanation:
Gardener´s syndrome or familial adenomatous polyposis is characterized by multiple small and large intestinal tumours and lipomas. Osteomata and fibromas are also seen.
It is a rare familial condition, when arising from mutations of the APC gene is inherited in an autosomal dominant fashion.
Gardener´s syndrome carries an increased risk of papillary carcinoma of the thyroid with preponderance for female patients. -
Question 21 of 100
21. Question
1 pointsA 36-year-old female from Brazil presents to her physician with complaints of palpitations, shortness of breath, and orthopnea for several weeks. She also has symptoms of bloating, heartburn, and dysphagia. There is no associated fever or chills. Her chest X-ray reveals an enlarged heart. What is the diagnostic test of choice to make the diagnosis?
Correct
Incorrect
Explanation:
This patient has Chagas´ disease, which is endemic to Central American countries.
Infection with Trypanosoma cruzi is transmitted via reduviid bugs and can cause myocarditis, leading to congestive heart failure in its chronic stages.
Diagnosis of acute Chagas´ disease is by examination of the peripheral smear. Diagnosis of chronic Chagas´ disease is by immunofluorescence testing and ELISA. Echocardiogram would only tell us that the patient has cardiomegaly and a low ejection fraction, but not the cause of congestive heart failure. The barium swallow would diagnose achalasia or dilated esophagus secondary to Chagas´ disease. This would cause symptoms of dysphagia. Peripheral smear examination is useful to diagnose acute Chagas´ disease. Acute phase is generally limited to facial edema and nodule (chagoma) near the bite coupled with fever, lymphadenopathy, and hepatosplenomegaly. Cardiac manifestations are associated with the chronic form of the disease in which few trypomastigotes are found in the blood. The electrocardiogram may reveal conduction defects, but the specific diagnosis of Chagas´ disease cannot be made from the electrocardiogram. -
Question 22 of 100
22. Question
1 pointsWhat is the most common lesion responsible for diabetic nephropathy?
Correct
Incorrect
Explanation:
Interstitial nephritis can cause diabetic nephropathy but is much less common than diffuse glomerulosclerosis. IgA deposition and glomerulonephritis are unusual in diabetes. Nephrolithiasis may be associated with diabetic nephropathy but is much less common than.
-
Question 23 of 100
23. Question
1 pointsThe thalassemias are caused by which one of the following?
Correct
Incorrect
Explanation:
The thalassemia family is characterized by a congenital disorder in synthesis of one or more of the subunits of hemoglobin. Increased hemolysis and splenic sequestration may be found as a result of abnormal red cells, but they are not causes of thalassemia. Thalassemia is not caused by blood loss or sulfonamides.
-
Question 24 of 100
24. Question
1 pointsMultiple endocrine neoplasia type I syndrome is NOT associated with
Correct
Incorrect
Explanation:
Medullary thyroid carcinoma is not associated with multiple endocrine neoplasia syndrome type I, but is associated with type II. The other choices are all associated with type I.
-
Question 25 of 100
25. Question
1 pointsA 7 year old girl walking across a vacant lot steps on a nail. Next day, her foot is sore and the wound appears inflamed. During these early stages of infection, leukocytes leave the circulation to enter the damaged tissues following a concentration gradient. The molecule that would be most important in this process is
Correct
Incorrect
Explanation:
The most important chemotactic factors for neutrophils are the complement factor C5a and the interleukin-8.
The cytokines IL-1 and tumor necrosis factor have complex similar actions including stimulation of production of many acute-phase reactions, stimulation of fibroblasts, and stimulation of endothelium. Leukotrienes LTC4 and LTD4 cause increased vascular permeability. Prostaglandins PGI2 and PGD2 mediate vasodilation and pain. Thromboxane and platelet activating factor promote vasoconstriction and platelet aggregation. -
Question 26 of 100
26. Question
1 pointsLymph node biopsy of a 48 year old woman, who presents with fatigue and painful lymph nodes in her neck for 6 months, reveals hypercellularity in the cortical areas; and serum electrophoresis shows a spike of protein in the gamma region. The abnormal immunoglobulin is IgG2 isotype. What is the composition of an IgG2 molecule?
Correct
Incorrect
Explanation:
IgG molecules contain two gamma heavy chains of a given subtype and two light chains (either kappa or lambda). The 2 in IgG2 indicates the subclass to which the molecule belongs. IgG2 contains two gamma2 chains (since a given B cell can only form one type of heavy chain). The IgG molecule will contain either two kappa chains or two lambda chains, but never one of each. One alpha, one gamma2, and two kappa chains is not correct, because a given cell produces immunoglobulin molecules with a single type of heavy chain. One gamma1 chain and two kappa chains is not correct. Each immunoglobulin has two heavy chains and two light chains, so this option lacks a heavy chain to make a complete g IgG 1 molecule. Two gamma1 chains and one kappa and one lambda chain is not correct; because an immunoglobulin molecule must contain two identical heavy chains and two Identical light chains. Two gamma1 chains and two kappa chains is not correct, since this describes a complete IgG1 molecule.
-
Question 27 of 100
27. Question
1 pointsA 46 year old man has gradual weight gain over the past several years. His fingers have enlarged so much that he can no longer wear his wedding rind. He sweats more than usual, and in particular, his hands are constantly sweaty. He has also noticed a gradual coarsening of his facial features. Which endocrine abnormality is likely to be present?
Correct
Incorrect
Explanation:
The patient probably has acromegaly due to a growth hormone secreting adenoma in the anterior pituitary. Hypersecretion of growth hormone in an adult will not cause an (increase in stature, since the epiphyses of long bones have already fused. However overgrowth of bone in the face and skull produces the characteristic protruding jaw and forehead observed in this disorder. Soft-tissue proliferation leads to a coarsening of facial features. The hands and feet are particularly affected, producing large, thickened, spade like fingers and toes. Excessive growth hormone decreases the sensitivity of peripheral tissues to insulin (“anti-insulin” effect). This tends to raise blood glucose and produce a compensatory hyperinsulinemia (not decreased plasma insulin) that functions to limit the hyperglycemia. Approximately 50% of patients with acromegaly show impaired glucose tolerance. Plasma levels of growth hormone and IGF-1 are both increased in acromegaly. Administration of oral glucose does not suppress growth hormone in patients with acromegaly as it does in normal individuals.
-
Question 28 of 100
28. Question
1 pointsA female with G6PD deficiency develops a urinary tract infection. Free radicals generated by responding neutrophils create an oxidant stress, and RBCs are rapidly over whelmed. Serum levels of which substance decrease as soon as intravascular hemolysis begins?
Correct
Incorrect
Explanation:
Haptoglobin (or alpha 2globulin) is a serum protein that promptly binds to tree hemoglobin in the serum, forming a tight complex. The haptoglobin hemoglobin complex is quickly cleared from the blood by the reticuloendothelial system, which allows the hemoglobin to be salvaged instead of lost into the urine. Intravascular hemolysis from any cause is rapidly followed by a dramatic drop in serum haptoglobin. Bilirubin is synthesized in the liver and spleen as an end-product of heme degradation. Serum bilirubin levels would not change until the haptoglobin in the blood was completely saturated, and tree hemoglobin began to circulate. In this circumstance, serum bilirubin would increase, not decrease. C reactive protein is an acute phase reactant produced by the liver in increased quantity in response to inflammation. This patient´s urinary tract infection would increase C reactive protein levels, but the hemolysis would not further affect them. Serum free iron does not change appreciably in intravascular hemolysis. Ferrous iron in the red blood cell is complexed to hemoglobin, and this complex is not broken in hemolysis. Methemoglobin (an oxidized form of hemoglobin) increases in the blood along with free hemoglobin in cases of intravascular hemolysis
-
Question 29 of 100
29. Question
1 pointsA 71 year old woman presents to the doctor prior to beginning chemotherapy for newly diagnosed small cell lung carcinoma. Exam is notable for obesity, BP of 180/110 mm Hg, facial hair, abdominal striae, and an acneiform rash on her chest and back. Serum glucose is 250 mg/dL. CXR shows a right perihilar mass and severe diffuse osteoporosis. This clinical picture is likely due to
Correct
Incorrect
Explanation:
This woman has all the classic findings of Cushing syndrome: obesity, hypertension, hirsutism, acne, striae, glucose intolerance, and osteoporosis. Cushing syndrome may be caused by excess production of cortisol due to bilateral adrenal hyperplasia or an adrenal neoplasm; by excess production of corticotropin (ACTH) by a pituitary adenoma; or by ectopic production of ACTH by a tumor, most commonly a small cell lung carcinoma (major clue in the question stem!). Destruction of the adrenal glands bilaterally or of the anterior pituitary by metastases would cause a deficiency of cortisol and ACTH, respectively, and would lead to a syndrome of cortisol deficiency with orthostatic hypotension, malaise, nausea, and weight loss. Ectopic production of gastrin, as seen in Zollinger-Ellison syndrome, causes severe refractory peptic ulcer disease.
Ectopic production of parathyroid hormone (PTH), which can be seen in squamous cell lung carcinoma, would result in hypercalcemia. -
Question 30 of 100
30. Question
1 pointsWhich one of the following statements regarding X linked diseases is correct?
Correct
Incorrect
Explanation:
X linked recessive diseases can occur with the same severity in females in certain situations, having an affected father and carrier mother (seen in G6PD deficiency), Turner´s syndrome, X chromosome isodisomy, unequal lyonisation, in an XY individual with testicular feminisation syndrome and where there is an X/autosome translocation through a gene (DMD).
Fragile X syndrome shows anticipation with triplet repeat expansion. Y linked diseases can be associated with male infertility.
There are several X linked immune deficiency syndromes and severe combined immune deficiency, chronic granulomatous disease, agammaglobulinaemia (Bruton´s disease), and Wiscott-Aldrich syndrome. Hypophosphataemic rickets is X linked dominant, and Von Willebrand´s disease is inherited in an autosomal dominant manner. -
Question 31 of 100
31. Question
1 pointsApoptosis is induced by:
Correct
Incorrect
Explanation:
A key event in the initiation of apoptosis in the activation of a cascade of cysteine-aspartate specific proteases known as caspases.
-
Question 32 of 100
32. Question
1 pointsWhich one of the following diseases is associated with an increased risk of colon cancer?
Correct
Incorrect
Explanation:
People with inflammatory bowel disease (ulcerative colitis and Crohn´s disease) are at increased risk of colon cancer. The risk is greater the longer a person has had the disease. The other conditions are not associated with a predisposition to colon cancer
-
Question 33 of 100
33. Question
1 pointsVon Willebrand´s disease is the most common inherited bleeding disorder. All of the following are true of von Willebrand´s factor EXCEPT.
Correct
Incorrect
Explanation:
Von Willebrand´s disease, the most common inherited bleeding disorder, is reported to occur in up to one out of every 1,000 persons. Von Willebrand´s factor is a heterogeneous multimeric plasma glycoprotein with two major functions: it facilitates platelet adhesion by linking platelet membrane receptors to vascular subendothelium, and it serves as a plasma carrier for factor VIII. In the classic type I form, inheritance is autosomal dominant. In the much less common type II variant, there is a qualitative rather than a quantitative defect in von Willebrand´s factor. In type II disease, due to these qualitative defects, clinical bleeding may occur in the presence of normal or only slightly decreased levels of von Willebrand´s factor or factor VIII.
-
Question 34 of 100
34. Question
1 pointsA 54 year old male was diagnosed as having hereditary spherocytosis. One of his children was also shown to have spherocytosis. Which of the following is FALSE regarding hereditary spherocytosis?
Correct
Incorrect
Explanation:
Thalassemia major, not hereditary spherocytosis, affects red blood cells by decreasing the production of the beta globin chains of hemoglobin and causing excess free alpha chains to accumulate in the erythrocytes, resulting in a decrease in membrane plasticity but also in premature loss of membrane by shedding. The consequent decrease in the ratio of surface area to cellular volume causes the red cell to change from a biconvex disk to a sphere. Splenectomy results in an improvement in the anemia and a decrease in the reticulocyte count. A postsplenectomy blood smear would still reveal an abundance of spherocyres, but these cells would not be trapped by splenic sequestration. Congenital abnormalities in the red blood cell membrane structure results in a rigid membrane, predisposing the red blood cell to sequestration in the splenic cords, and, thus, anemia. Multiple gallstones and symptomatic anemia are both reasons to undergo a splenectomy in heredity spherocytosis. Cholecystectomy is indicated, as well, in symptomatic cholelithiasis.
-
Question 35 of 100
35. Question
1 pointsOut of the following, which finding is consistent with a diagnosis of complicated atheroma?
Correct
Incorrect
Explanation:
Uncomplicated atheromas include smooth muscle, collagen, elastin, and proteoglycans within the plaque. Although they appear yellow when cut, they are grayish white in morphologic appearance. Calcifications and hemorrhage do not occur in uncomplicated, only complicated, atheromas.
-
Question 36 of 100
36. Question
1 points- A 57 year old homeless man is brought to the ER. His temperature is 38.8°C (101 .8°F) rectally and he smells of cheap wine. Examination shows dullness to percussion, increased vocal fremitus, and decreased breath sounds and wet crackles on the right. CXR reveals consolidation of the right upper lobe. Production of thick, bloody sputum is also noted. The most likely cause is
Correct
Incorrect
Explanation:
This man has pneumonia caused by Klebsiella pneumoniae as suggested by his alcoholism, lobar consolidation, and “currant jelly” sputum. Klebsiella is a member of the family Enterobacteriaceae, so it is a gram negative, oxidase negative bacillus. A gram negative, oxidase positive bacillus describes Pseudomonas aeruginosa, which may cause pneumonia but would be more likely in a patient with cystic fibrosis than in an alcoholic. The sputum would have a blue-green color and a fruity odor. A gram negative pleomorphic bacillus requiring factors X and V describes Haemophilus influenzae. It is a cause of pneumonia, pharyngitis, bronchitis, and otitis media, but does not typically cause lobar consolidation or bloody sputum. A gram positive, catalase negative coccus describes the genus Streptococcus, of which S. pneumoniae is the most common cause of typical pneumonia in adults (including alcoholics). The sputum of streptococcal pneumonia, however, is “rusty,” or tinged with brown blood, not bright red and gelatinous as described here. A gram positive, catalase positive coccus describes the genus Staphylococcus, of which S. aureus is a common cause of pneumonia. It is most commonly associated with aspiration pneumonia in alcoholics, but does not cause bloody sputum.
-
Question 37 of 100
37. Question
1 pointsA 7 year old child develops edema, hypertension and hematuria several weeks after a severe streptococcal sore throat. Best description of the properties of the most likely infectious organism is given by which one of the following?
Correct
Incorrect
Explanation:
The most likely cause of glomerulonephritis following sore throat is Streptococcus pyogenes. This is a Group A streptococcus, which is characterized by beta hemolysis and inhibition by the antibiotic bacitracin. Streptococcus pyogenes pharyngitis can also be followed by rheumatic fever. Streptococcus pneumoniae is an example of an alpha hemolytic streptococcus that is inhibited by optochin and lysed by bile. Streptococcus mutans is an example of an alpha hemolytic streptococcus that is not inhibited by Optochin nor lysed by bile. Enterococcus faecalis is an example of a beta hemolytic streptococcus that can grow in 40% bile and 6.5% sodium chloride. Streptococcus agalactiae is an example of a beta hemolytic streptococcus that is resistant to bacitracin and positive for the CAMP test (an incomplete hemolysin).
-
Question 38 of 100
38. Question
1 pointsA 25 year old male has itching on his arms and face. Examination shows well circumscribed wheals with raised, erythematous borders and blanched centers. The type of hypersensitivity that is this patient most likely having is
Correct
Incorrect
Explanation:
Urticaria (hives) is a good example of a local anaphylaxis reaction which is classified as a type I hypersensitivity reaction. Type I hypersensitivity reactions involve preformed IgE antibody bound to mast cells or basophils, which release vasoactive and spasmogenic substances when they react with antigens. The eosinophil-mediated cytotoxicity against parasites is an example of antibody-dependent, cell-mediated cytotoxicity. Type II hypersensitivity is caused by autoantibodies against cells or tissues. When these antibodies activate complement or promote phagocytosis, the tissue is damaged. Type III hypersensitivity is caused by immune complex deposition in the small diameter blood vessels and the subsequent activation of complement and destruction of the vasculature. The tuberculin (PPD) reaction, used to test for tuberculosis exposure, is an example of delayed-type, or type IV, hypersensitivity.
-
Question 39 of 100
39. Question
1 pointsA 66 year old female treated 12 years ago with aggressive chemotherapy and radiation for a melanoma of the eye is found to have multiple metastatic liver foci of this tumor. Now her oncologist suggests trying an immunologic approach. A population of her peripheral blood leukocytes are harvested and cultured in vitro to produce lymphokine-activated killers (LAK) which are then re-infused into her circulation. The cytokine that is used to produce LAK cells is
Correct
Incorrect
Explanation:
LAK cells are natural killer cells that have been activated in vitro by culture with IL-2.This cytokine which is produced by T helper cells, causes activation and cloning of many categories of lymphocytes. IL-1 is endogenous pyrogen. It is a product of activated macrophages and causes fever by affecting the hypothalamic temperature set-point. It would not cause a beneficial effect on cytotoxic cells if used in an in vitro culture. IL-3 is a cytokine that enhances the production of myeloid cells in the bone marrow. It would not cause a beneficial effect on cytotoxic cells lf used in an in vitro culture. IL-4 is a product of TH2 cells that stimulates isotype switching to IgE in B lymphocytes. It would not cause a beneficial effect on cytotoxic cells lf used in an in vitro culture. IL-5 is a product of TH2 cells that stimulates isotype switching to IgA in B lymphocytes. It would not cause a beneficial effect on cytotoxic cells lf used in an in vitro culture.
-
Question 40 of 100
40. Question
1 pointsA 4 year old girl´s physical examination is remarkable for extremity edema. Urinalysis shows significant proteinuria our no red cells or casts. Electron microscopy performed on a kidney biopsy would most likely reveal which finding?
Correct
Incorrect
Explanation:
The most frequent cause of nephrotic syndrome in children is minimal change disease (Iipoid nephrosis), which is characterized by an absence of findings by light microscopy and by fusion of epithelial toot processes on electron microscopy. Dense deposits are seen in type II membranoproliferative glomerulonephritis. Mesangial deposits are a prominent feature of IgA nephropathy and are seen to a lesser degree in several other glomerulonephritides. Subendothelial deposits are seen in type I membranoproliferative glomerulonephritis. Subepithelial spikes are a feature of membranous glomerulonephritis.
-
Question 41 of 100
41. Question
1 pointsA brain tumor biopsy of a 42 year old male shows a glial neoplasm consisting of atypical astrocytes with scattered mitoses. Besides mitotic activity, which markers can provide information about neoplasm´s proliferative activity?
Correct
Incorrect
Explanation:
Ki-67 is a nuclear factor (of uncertain function) whose expression correlates with neoplastic replicative activity. Its expression can be visualized by immunostaining of formalin fixed, paraffin-embedded sections. Ki-67 labeling correlates with a neoplasm´s rate of growth and, therefore, with prognosis. The bcl 2 gene suppresses apoptosis by different mechanisms. Its abnormal activation is involved in the pathogenesis of low-grade lymphomas but not astrocytomas. GFAP is an intermediate cytoskeletal filament (analogue of keratin and vimentin) expressed exclusively by certain types of glial cells, e.g. astrocytes and ependymal cells. Immunohistochemistry for GFAP is mod diagnostically to confirm an astrocytic origin of a neoplasm, but gives no information about mitotic or proliferative rate. The gene p53 encodes a protein that blocks the cell cycle when damage to DNA occurs. If the damage is successfully repaired, p53 allows the cell cycle to resume; if not, p53 induces apoptosis, thus eliminating dangerous DNA mutations. Mutations of p53 have been found in the great majority of human neoplasms, including gliomas. However, its expression gives no information concerning neoplastic replicative activity. Ubiquitin is a low molecular weight heat shock protein. Its function is to tag aberrant proteins for degradation. It is present in many abnormal intraneuronal inclusions associated with neurodegenerative disorders, such as Lewy bodies, Pick bodies, and neurofibrillary tangles. It has no relationship with mitotic activity or growth rate.
-
Question 42 of 100
42. Question
1 pointsA 78 year old man is recovering 10 days after a massive anterolateral infarction of the left ventricle. He suddenly develops paralysis of the entire right half of the body, including facial and eye muscles. What complication of MI is likely to have precipitated this event? Correct
Incorrect
Explanation:
This patient has suffered a cerebrovascular accident as a consequence of thromboemboli emanating from a mural thrombus formed over the recent myocardial infarction. Mural thrombus frequently develops over a previously infarcted segment of myocardium, especially when the infarction is large and a ventricular aneurysm develops. Ischemic damage to the endocardium, soluble factors released by the injured myocardium, and altered wall kinetics that produce sluggish blood flow, all favor mural thrombus formation. One important consequence of a mural thrombus is thromboembolism to the systemic circulation, producing a stroke, as in this patient´s case. Calcific coronary atherosclerosis is yew likely responsible for the patient´s original myocardial infarction. Coronary atherosclerosis is not a consequence of myocardial infarction, and it does not directly predispose to the development of embolic strokes. Complicated atherosclerotic lesions in the carotid or cerebral circulation, however, may directly lead to a cerebrovascular accident. Electromechanical dissociation (EMD) is a catastrophic event that frequently leads to sudden death, not stroke. In EMD although a normal potential is transmitted through the myocardium, no pulse of arterial blood is sent to the systemic circulation. EMD may occur as a consequence of pericardial tamponade, massive pulmonary embolism, or myocardial toxins that prevent normal cardiac muscle contraction. Left bundle branch block represents failure of electrical transmission along the Purkinje fibers to the left ventricular myocardium. Bundle branch blocks are important causes of arrhythmia, but are not responsible for cerebrovascular accidents. Myocardial rupture is an infrequent consequence of myocardial infarction that typically occurs approximately 1 week after the infarction. If scarring of the infarcted segment does not keep pace with necrosis and digestion of the infarcted tissue, the myocardium can rupture under the high intraventricular pressures, and pericardial tamponade generally ensues. Myocardial rupture rapidly leads to acute heart failure, not stroke.
-
Question 43 of 100
43. Question
1 pointsA girl aged 10 years presents with sore throat and fever. She denies any cough or rhinorrhea. Throat culture grows bacitracin sensitive bacterial colonies. The microorganism would be protected from the lytic action of detergents by its
Correct
Incorrect
Explanation:
This girl has streptococcal pharyngitis. The infecting organism is group A beta-hemolytic streptococcus (Streptococcus pyogenes); its growth is inhibited by the placement of a bacitracin disk on the throat culture plate. (Beta-hemolysis occurs as the result of the bacterial hemolysin streptolysin S.) This is a gram-positive bacterium and therefore possesses a very thick peptidoglycan layer that would protect it from lysis by detergents. (Note that gram positives also contain teichoic acid.) In contrast, gram negatives have a thin peptidoglycan layer. Keratin-like proteins in the spore coat and calcium ion chelators (dipicolinic acid) are found in spores formed by species of Bacillus and Clostridium. These protect the spores from the elements: dehydration, heat, chemicals, radiation, etc. Lipopolysaccharide in the outer membrane and a periplasmic space are found in gram-negative organisms. The lipopolysaccharide is an endotoxin, and the periplasmic space contains beta-lactamases in some species.
-
Question 44 of 100
44. Question
1 pointsA female with rheumatoid arthritis on long term corticosteroid therapy has nausea, vomiting, headache, and confusion. She has temperature of 103.4°F and nuchal rigidity. CSF analysis shows pleocytosis, increased protein, decreased glucose, and budding, encapsulated organisms. An India ink mount of the CSF sediment shows budding yeasts surrounded by a wide, clear zone. The most appropriate pharmacotherapy is
Correct
Incorrect
Explanation:
Amphotericin B is the most appropriate drug listed for the treatment of cryptococcal meningitis. It is a polyene antibiotic that binds to ergosterol in the fungal cell membrane, creating an artificial pore. Flucytosine is often prescribed as an adjunct medication. Fluconazole is used long-term to prevent recurrence in AIDS patients. Isoniazid inhibits the biosynthesis of mycolic acids in the mycobacterial cell wall. It is the primary drug used against tuberculosis. It is used alone for TB prophylaxis and in combination with other antituberculars to treat patients with active disease. Ketoconazole is an orally administered imidazole antifungal medication. It inhibits 14 alpha demethylase to block the synthesis of fungal cell membrane ergosterol. Note the difference in mechanism between the polyenes, which alter ergosterol structure, and the imidazoles, which block ergosterol synthesis. Ketoconazole is often used to treat coccidioidomycosis, histoplasmosis, blastomycosis, paracoccidioidomycosis, and mucocutaneous candidiasis. Metronidazole is an antiprotozoal drug useful in treating a variety of parasitic infections. It is the drug of choice for trichomoniasis and giardiasis and provides general anaerobic coverage. This makes it useful for treating postsurgical abdominal and pelvic Bacteroides fragilis infections or flare ups of intestinal diverticulitis. Nystatin is an antifungal polyene that is usually used topically but can be taken orally for oral and esophageal candidiasis. Candidal infections of the skin, mucous membranes, and vagina usually respond well to this drug. It may also be used to prevent intestinal fungal overgrowth in patients on chemotherapy.
-
Question 45 of 100
45. Question
1 pointsA 55 year old man has gnawing pain in the mid epigastrium, with occasional radiation to the back. He has lost 15 pounds over the past 3 months. Pancreatic carcinoma is suspected. The tumor marker would aid in confirming the diagnosis is
Correct
Incorrect
Explanation:
Tumor markers can be very helpful in narrowing the possible primary sources for metastatic lesions. Carcinoembryonic antigen (CEA) can be seen in any tumor derived from gut epithelium, notably colon cancer and pancreatic cancer. Alpha-fetoprotein (AFP) is seen in hepatocellular carcinoma, embryonal cell tumor of the testis, and malignant teratoma. CA-125 is produced by ovarian cancer. The beta subunit of human chorionic gonadotropin (hCG) is seen in choriocarcinoma, hydatidiform mole, and germinoma. Prostate-specific antigen (PSA) is seen in pro-static carcinoma. You should also remember that serum elevations of many of these markers can also be seen in some benign conditions of similar tissues. hCG, for example, is elevated normally in pregnancy.
-
Question 46 of 100
46. Question
1 pointsA 55 year old woman with chronic microcytic hypochromic anemia also has a sore, smooth, red tongue and a sense of dysphagia midway during swallowing. She is at increased risk for developing which condition?
Correct
Incorrect
Explanation:
The patient has Plummer Vinson syndrome, characterized by atrophic glossitis, esophageal webs, and iron-deficiency anemia. Plummer Vinson is usually located in the proximal third of the esophagus or in the hypopharynx. Patients with this syndrome are at increased risk of developing squamous cell carcinoma of the esophagus. This question is testing your knowledge not only of the disease but also of its complications. Remember that adenocarcinomas occur in the distal one-third of the esophagus, therefore you must also know that squamous cell carcinomas occurs in the proximal two-thirds of the esophagus. Barrett esophagus and adenocarcinoma of the esophagus are associated with reflux esophagitis. Candida and CMV, esophagitis can be seen in immunosuppressed patients, including patients with AIDS. -
Question 47 of 100
47. Question
1 pointsA 48 year old male is brought after severing a major artery during a farm accident. It is estimated that the patient lost about 800 mL of blood. His blood pressure is 95/65 mm Hg. Decrease in which one of the following would be expected in response to hemorrhage in him?
Correct
Incorrect
Explanation:
The decrease in blood pressure caused by hemorrhage activates the baroreceptor reflex, which tends to increase sympathetic nerve activity and decrease parasympathetic (vagal) nerve activity. The fact that the patient has lost 800 mL of blood and yet his blood pressure has decreased only slightly may be attributed to the following compensatory responses: baroreceptor reflex, chemoreceptor reflex, epinephrine and norepinephrine released from the adrenal medulla, formation of angiotensin II formation of vasopressin, and the shift of fluid from the tissues into the capillaries. The increase in heart rate that occurs during hemorrhage can be attributed to decreased vagal tone and increased sympathetic nerve activity. Plasma renin activity is increased during hemorrhage. Activation of the renin angiotensin system during hemorrhage plays an important role in maintaining blood pressure. Angiotensin II increases blood pressure acutely by constricting arterioles throughout the body, and chronically by decreasing the renal excretion of both salt and water. The increase in sympathetic nerve activity constricts blood vessels throughout the body, which causes the total peripheral resistance to increase. -
Question 48 of 100
48. Question
1 pointsA 42 year old woman has a recent onset of fatigue, malaise, constipation, and a 12 lbs weight gain. On examination, her thyroid is firm and enlarged. Which lab data is likely to confirm the expected diagnosis?
Correct
Incorrect
Explanation:
The patient´s presentation is consistent with hypothyroidism. Serum thyroid stimulating hormone (TSH) measurement is most likely to confirm the empiric diagnosis. TSH levels usually rise above normal before serum thyroxine (T4) and serum triiodothyronine (T3) levels do even in mild cases of hypothyroidism. Therefore, TSH measurement would be the most accurate test to determine the presence of hypothyroidism regardless of the severity. A high titer of antithyroid antibodies is characteristic of chronic thyroiditis, which is the most common cause of hypothyroidism. However detection of these antibodies would not indicate it hypothyroidism was present. T3 resin uptake measurement is not an accurate test of thyroid function; it is primarily used to exclude various abnormalities in the thyroid hormone binding proteins. -
Question 49 of 100
49. Question
1 pointsA 42 year old woman has a recent onset of fatigue, malaise, constipation, and a 12 lbs weight gain. On examination, her thyroid is firm and enlarged. Which lab data is likely to confirm the expected diagnosis?
Correct
Incorrect
Explanation:
The patient´s presentation is consistent with hypothyroidism. Serum thyroid stimulating hormone (TSH) measurement is most likely to confirm the empiric diagnosis. TSH levels usually rise above normal before serum thyroxine (T4) and serum triiodothyronine (T3) levels do even in mild cases of hypothyroidism. Therefore, TSH measurement would be the most accurate test to determine the presence of hypothyroidism regardless of the severity. A high titer of antithyroid antibodies is characteristic of chronic thyroiditis, which is the most common cause of hypothyroidism. However detection of these antibodies would not indicate it hypothyroidism was present. T3 resin uptake measurement is not an accurate test of thyroid function; it is primarily used to exclude various abnormalities in the thyroid hormone binding proteins.
-
Question 50 of 100
50. Question
1 pointsA 42 year old man has daily burning epigastric pain of 4 months duration. He is sometimes awakened at night by coughing spells. He also has has diarrhea and has lost 10 pounds in the last 2 months. Investigations are significant for mild hypercalcemia and endoscopy reveals small duodenal ulcers. Antral biopsy is negative for H. pylori. What is the likely diagnosis?
Correct
Incorrect
Explanation:
Zollinger Ellison syndrome results from hypersecretion of gastric acid due to a gastrin secreting neoplasm (gastrinoma). The esophageal symptoms, epigastric pain, and duodenal ulcers are all due to excessive acid secretion. Diarrhea occurs in about half of Zollinger Ellison patients. It is partly due to the excess gastric secretions delivered to the intestines, and partly due to maldigestion because the low pH in the duodenal lumen creates a poor environment for function of pancreatic enzymes. Furthermore the low duodenal pH can damage the intestinal mucosa and flatten the villi, leading to malabsorption and weight loss. Typically, the patients present with elevated basal acid output, markedly elevated gastrin levels, and a positive secretin stimulation test. A small percentage of cases are associated with multiple endocrine neoplasia type 1 (MEN-1), which is characterized by parathryroid, pituitary, and pancreatic tumors. This would explain the mild hypercalcemia of this patient. Most often the plasma concentration of gastrin is not significantly increased with classic duodenal ulcers. In addition, H. pylori is present in over 90% of duoduenal ulcers, and multiple ulcers are less common than in Zollinger Ellison syndrome. The endoscopic examination failed to provide evidence of a gastric ulcer. Furthermore, the incidence of H. pylori infection is very high with gastric ulcer (70-80%). Patients with gastric ulcer often exhibit decreased basal acid secretion and the ulcer is usually due to breakdown of the gastric mucosal barrier. Type A chronic gastritis is also a cause of increased circulating gastrin. In this case, the autoimmune destruction of the parietal cells leads to achlorhydria. A major factor that normally suppresses acid secretion by the parietal cells is the low pH of gastric juice. With achlorhydria, the pH of the juice rises and gastrin secretion increases (it can be as high as in Zollinger Ellison syndrome). However, with chronic gastritis the basal secretion of acid would be decreased, not increased. Furthermore, multiple duodenal ulcers are unlikely. A vasoactive intestinal peptide-secreting tumor (VIPoma) produces a watery diarrhea because of excessive cAMP dependent secretion of chloride and water by intestinal cells in the crypts of Lieberkühn. However, the symptoms of dyspepsia, presence of duodenal ulcers, and hypersecretion of acid and gastrin are not present. In fact, the excessive vasoactive intestinal peptide usually suppresses gastric acid secretion.
-
Question 51 of 100
51. Question
1 pointsA 58 year old type 2 diabetic woman has burning during urination and a fever. A urinalysis is ordered with culture and sensitivity. She has had four urinary tract infections in the past year. At a follow up the physician learns that he has been accidentally wetting herself. A consequence of diabetes that would likely cause this symptom is
Correct
Incorrect
Explanation:
Diabetic neuropathy is a poorly understood phenomenon characterized by damage to Schwann cells and axons in the spinal cord and especially, the peripheral nerves. The peripheral neuropathy is symmetrical and favors the lower extremities. Autonomic neuropathy is most likely to produce sexual, bowel or bladder dysfunction. Bladder prolapse is a common cause of urinary incontinence in older women with weakened pelvic floor musculature due to the effects of aging and prior pregnancies. Glomerulosclerosis a very frequent complication of diabetes mellitus, leads to proteinuria and renal failure, but does not predispose an individual to urinary incontinence. Normal pressure hydrocephalus is a slowly progressive, nonobstructive hydrocephalus that characteristically occurs in the elderly and produces incontinence, gait disturbance and cognitive changes. Its cause is not known, but it is nota direct consequence of diabetes. Toxic myopathies represent direct chemical damage to skeletal muscle due to recognized chemical toxins. Examples include thyrotoxicosis, and alcohol and drug related myopathies. There is no identified myopathy associated with diabetes mellitus.
-
Question 52 of 100
52. Question
1 pointsA 2 year old child´s radiograph reveals a new fracture of the humerus and evidence of multiple old fractures in ribs and long bones of the extremities. He shows minimal bruising. Careful examination reveals that he has “peculiar teeth,” a blue tinge to the sclera, and unusually mobile joints. A disease is suspected that is characterized by an abnormality of which biochemical function?
Correct
Incorrect
Explanation:
The child has the most common variant (type I) of osteogenesis imperfecta, which is an autosomal dominant genetic defect in the synthesis of type I collagen, due to decreased synthesis of the procollagen alpha 1 amino acid chain. This defect (unlike that of the perinatal, lethal, type II form of osteogenesis imperfecta) is compatible with survival but does cause skeletal fragility, dentinogenesis imperfecta (abnormal teeth), blue sclera, joint laxity, and hearing impairment. Unfortunately, a number of children with this defect have been removed from their parents because of “abuse,” only to have the broken bones continue in the new environment. Type I collagen is found in skin, bone, tendons, and most other organs. Type II collagen is found in cartilage and vitreous humor. Type III collagen is found in blood vessels, uterus, and skin. Type IV collagen makes basement membranes. Type V collagen is a minor component of interstitial tissues and blood vessels. There are also type VI-XI collagens, which are minor constituents of various tissues.
-
Question 53 of 100
53. Question
1 pointsA 24 year old construction worker falls down on his outstretched right hand. X-ray shows a fracture of one of the wrist bones. The physician is concerned about the risk for avascular necrosis. The wrist bone that was fractured is
Correct
Incorrect
Explanation:
Suspect fracture of the scaphoid (navicular) bone in any young adult who has “fallen on an outstretched hand” (classic clue). The physician was concerned about the risk of avascular necrosis because, in some people, the blood supply of the bone is located distally, and a fracture will deprive the proximal region of the bone of its arterial nourishment. Osteoarthritis is a common complication of scaphoid fractures that do not heal properly. The lunate bone is commonly dislocated in patients who have fallen on an outstretched hand. Median nerve injury is a frequently associated occurrence. The capitate, pisiform, and trapezoid bones of the wrist are less commonly associated with falls on an outstretched hand.
-
Question 54 of 100
54. Question
1 pointsA 62 year old male has difficulty arising from a chair and initiating new movements. On exam, there is resting hand tremor and cog wheel rigidity. Which amino acid is the precursor for the neurotransmitter that is deficient in his brain?
Correct
Incorrect
Explanation:
This question requires three steps of logic. First, figure out the diagnosis (a classic case of Parkinson disease), then remember which neurotransmitter is involved in the disease (dopamine), and, finally, recall which amino acid serves as the precursor for that neurotransmitter (tyrosine). The hydroxylation of tyrosine by tyrosine hydroxylase results in DOPA, which is then decarboxylated to dopamine. Note that norepinephrine and epinephrine are also tyrosine derivatives, as are the melanins and the thyroid hormones thyroxine and triiodothyronine. Glutamate can be converted to the inhibitory neurotransmitter GABA by the action of glutamate decarboxylase. Glycine is involved in the synthesis of both creatine (along with arginine and S-adenosylmethionine) and heme (along with succinyl CoA). Histidine can be decarboxylated to histamine, an important inflammatory mediator. Tryptophan can be converted to serotonin by a hydroxylation reaction (tryptophan hydroxylase) followed by a decarboxylation reaction.
-
Question 55 of 100
55. Question
1 pointsA 65 year old man with atherosclerosis suffers an embolic stroke that leaves him with a left leg paresis. Exam shows a Babinski sign on the left and diminished sensation over his left leg. Which vessel blockade is responsible for his symptoms?
Correct
Incorrect
Explanation:
Three pieces of information are needed to answer this question. First, a lesion of the right motor cortex leads to a left paresis or paralysis. Second, the leg is represented in the part of the motor cortex that is adjacent to the interhemispheric fissure. Third, the anterior cerebral artery supplies the medial surface of the hemisphere extending from the frontal pole to the parieto-occipital sulcus; this would include the motor and somatosensory cortices. Blockade of the left anterior cerebral artery would lead to a right leg paresis or paralysis and diminished sensation over the right leg. The left middle cerebral artery supplies the lateral convexity of the right hemisphere. Blockade would lead to a paresis or paralysis of the right face and arm. The middle cerebral artery is the most common location for stroke. Blockade of the left posterior cerebral artery, which supplies the left occipital lobe, would lead to a right hemianopia with macular sparing.
The right middle cerebral artery supplies the lateral convexity of the right hemisphere. Blockade would lead to paresis or paralysis of the left face and arm. -
Question 56 of 100
56. Question
1 pointsA 31 year old otherwise healthy female has signs and symptoms of primary idiopathic pulmonary hypertension. Which pathologic finding would she show, either at autopsy or if an appropriate biopsy were taken?
Correct
Incorrect
Explanation:
This rare condition is thought to be related to the collagen vascular diseases, since up to 50% of patients have antinuclear antibodies (despite the absence of frank presentation of other autoimmune disease). Also, a similar, known secondary form of pulmonary hypertension is sometimes seen in patients with a wide variety of collagen vascular diseases, including systemic lupus erythematosus, polymyositis, dermatomyositis, systemic sclerosis, and adult and juvenile forms of rheumatoid arthritis. A wide variety of other conditions have also been associated with secondary pulmonary hypertension, including shunts, left atrial hypertension, chronic hypoxia, pulmonary embolism, drug reaction, hepatic cirrhosis, and sickle cell disease. Both primary and secondary forms of pulmonary hypertension are associated with prominent changes in the pulmonary vasculature, which can include muscularization of smaller arterioles, concentric hypertrophy of the intima (“onion skinning”), and a distinctive plexiform lesion (plexogenic pulmonary vasculopathy) in which the smallest arterioles become markedly dilated with lumens partially occluded by endothelial (or possibly mesenchymal) cells and sometimes thrombus. The prognosis of untreated pulmonary hypertension is poor. However, the use of the vasodilator hydralazine with anticoagulation can slow the course (fatal in about 3 years in untreated patients). If the pulmonary hypertension is secondary, therapy of the primary disease can be helpful. Unlike cor pulmonale, atrial fibrillation with mural thrombus formation is uncommon in primary pulmonary hypertension. The absence of left ventricular findings on echocardiography tends to exclude myocardial infarction as the source of the patient´s findings. The presence of enlargement of the main pulmonary artery excludes pulmonary artery stenosis. The clear lung fields exclude severe pulmonary fibrosis.
-
Question 57 of 100
57. Question
1 pointsA characteristic feature of primary hyperaldosteronism is which one of the following?
Correct
Incorrect
Explanation:
Primary hyperaldosteronism or Conn´s syndrome is characterised by hypokalaemic hypertension. Patients can present with tetany (alkalosis) and muscle weakness (hypokalaemia). Oedema and oliguria are more features of secondary hyperaldosteronism (cirrhosis); and vitiligo (suggesting auto-immunity) is not a feature.
-
Question 58 of 100
58. Question
1 pointsA 48 year old woman has a 6 month history of pruritus. Exam reveals jaundice, xanthelasma, scratch marks, vitiligo and 3 cm hepatomegaly. She was afebrile. LFTs reveal raised bilirubin alkaline phosphatase, gamma glutamyl transferase and mildly elevated alanine transaminase and aspartate transaminase. Which condition will most likely be found in her?
Correct
Incorrect
Explanation:
The most likely diagnosis is primary biliary cirrhosis as evidenced by pruritus, hypercholesterclaemia, jaundice, raised alkaline phosphatase (ALP) and gamma glutamyl transferase (γ-GT). Malsbsorption of fat soluble vitamins (A, D, K) is common.
-
Question 59 of 100
59. Question
1 pointsA 66 year old man presents with renal colic. The following day he passes a stone in his urine with analysis revealing that it is composed of uric acid. The most likely cause of this type of renal stone is which one of the following?
Correct
Incorrect
Explanation:
Uric acid stones occur in 5-25% of all cases of nephrolithiasis. They are associated with hyperuricaemia and hyperuricosuria. Predisposing factors for uric acid stone formation are dehydration, high purine lo (high protein diet), as a primary factor in idiopathic gout, and associated with high cell turnover (e.g. haematological malignancy). Chronic renal failure (B) is incorrect, as there is hyperuricaemia without hyperuricosuria. Hyperparathyroidism (C) is associated with calcium stones, not uric acid stones. Primary polycythaemia would predispose to uric acid stone formation, whereas secondary polycythaemia (E) does not. Allopurinol (A) is prescribed to treat gout and prevents uric acid formation; hence it reduces the frequency of uric acid Stones. The correct answer is thiazide diuretics (D). Thiazide diuretics cause hyperuricaemia and can predispose to hyperuricosuria and uric acid stone formation. Uric acid stones are also associated with underlying hypertension. Thiazide diuretics are used to treat calcium Stones, as they increase the reabsorption of calcium from the proximal tubules, preventing hypercalciuria.
-
Question 60 of 100
60. Question
1 pointsWhich of the following statements regarding phenylketonuria is correct?
Correct
Incorrect
Explanation:
Phenylketonuria (PKU) is the result of hyperphenylalaninaemia. It is autosomal recessive, with a prevalence of 1:10-20,000 live births. Phenylalanine is an essential amino acid. Dietary phenylalanine is not utilised for protein synthesis, but is normally degraded via the tyrosine pathway. Failure of this results in other metabolites that cause brain damage. Deficiency of phenylalanine hydroxylase (chromosome 12) or of the cofactor tetrahydrobiopterin (genes on chromosome 10 and 4) causes accumulation of phenylalanine in body fluids. The affected infant is usually normal at birth, and although blood phenylalanine levels may rise as early as four hours after birth, mental retardation develops gradually, and may not be apparent for a few months.
About 25% if infants have seizures, but over 50% have an abnormal EEG. Microcephaly, prominent maxilla growth retardation and wide-spaced teeth are found in untreated children. Even with dietary treatment, some degree of cognitive impairment is seen, and can vary from gross impairment or changes detected on cognitive tests.
Cerebral white matter changes are seen in older patients and may reflect a combination of late diagnosis and dietary indiscretion. Diagnosis of classic PKU requires raised Phe levels, normal plasma tyrosine levels, increased urinary Phe metabolites and normal cofactor (tetrahydrobiopterin) concentrations. -
Question 61 of 100
61. Question
1 pointsWhich of the following is a proto-oncogene?
Correct
Incorrect
Explanation:
Oncogenes are endogenous human deoxyribonucleic acid (DNA) sequences that arise from normal genes called proto-oncogenes. Proto-oncogenes are normally expressed in many cells, particularly during fetal development, and are thought to play an important regulatory role in cell growth and development. Alterations in the proto-oncogene can activate an oncogene, which produces unregulated gene activity, contributing directly to tumourogenesis. Oncogene alterations are important causes of:
- Rhabdomyosarcomas (ras oncogene)
- Burkitt´s lymphoma (C-myc is translocated intact from its normal position on chromosome 8 to chromosome 14)
- Neuroblastoma (N-myc proto-oncogene is seen in a proportion of patients with poor prognosis).
They should be contrasted with tumour suppressor genes. In this situation, the genes normally down regulate cell growth, and require inactivation to allow malignant growth. Examples include retinoblastoma.
-
Question 62 of 100
62. Question
1 pointsPolycystic ovarian disease is characterized by all of the following EXCEPT
Correct
Incorrect
Explanation:
Polycystic ovarian disease affects 3% to 6% of reproductive-age women. It is a heterogeneous disease that may be transmitted either as an autosomal dominant or X-linked trait. A variety of pathologic findings have been reported in the ovaries. The ovarian follicles have low aromatase activity. Adrenal androgens, which are elevated, may cause hirsutism. Anovulation results from FSH deficiency and LH excess.
-
Question 63 of 100
63. Question
1 pointsOut of the following, which finding is specifically pathognomonic of Wilson´s disease?
Correct
Incorrect
Explanation:
Kayser Fleischer rings, which represent deposits of copper in Descemet´s membrane of the cornea, are pathognomonic of Wilson´s disease. Hepatomegaly, renal insufficiency, anemia, and encephalopathy are all associated with Wilson´s disease, but are not specific for it.
-
Question 64 of 100
64. Question
1 pointsThe figure shows the genome of a strain of bacterium with resistance to multiple antibiotics. The loci encoding these resistances are shown. “IS” is the attachment site for a temperate phage. Which of the following resistance genes is most likely to be transferred by specialized transduction?
Correct
Incorrect
Explanation:
Specialized transduction is a mechanism of transfer of DNA that involves an accident in the life cycle of a temperate (lysogenic) phage. In cases in which lysogeny is terminated by the destruction of the viral repressor gene, errors of excision of the phage DNA out of the chromosome can cause transfer of bacterial DNA to another cell. The genes that are most likely to be transferred in this way are those that are closest to the insertion site of the phage. In this diagram, the drug resistance gene to “iS” is the sulfonamide resistance gene. Ciprofloxacin resistance is the gene located closest to oriT on this genetic map, so it is the gene that is most likely to be transferred during conjugation of this cell (an Hfr cell) with an F- cell.
Clindamycin resistance is the gene located downstream from the ciprofloxacin resistance gene, so it could be transferred in the Hfr to F- conjugation but it is drawn as being a long way away from oriT, so it is not as likely to be transferred in this way. Gentamicin resistance is the gene found on the other side of the insertion sequence for the phage, so it could be transferred by specialized transduction, but because it is shown as being farther away from “iS” than the sulfonamide resistance gene, it is not as likely to be transferred in this way. Vancomycin resistance is the gene located between oriT and tra. It is unlikely to be transferred during conjugation (that transfer could occur with ciprofloxacin resistance or clindamycin resistance, in that order). It is also unlikely that it would be transferred by, specialized transduction because it is at such a great genetic distance from the viral insertion sequence. -
Question 65 of 100
65. Question
1 pointsA 28 year old homozygous Rh D negative mother delivers her first child. The father is homozygous Rh D-positive. Child is born without any complications, but the mother is not given RhoGAM (anti-Rh IgG) after delivery. She delivers another child 2 years later who is anemic, is slightly jaundiced, and has an enlarged spleen and liver. The type of hypersensitivity that best describes this condition is which one of the following?
Correct
Incorrect
Explanation:
This is an example of type II hypersensitivity, in which antibody is produced against cells or receptors in the body. In this type of cytotoxic disease, the mother produced antibody against the Rh D-positive cells to which she was exposed upon delivers of her first child. This child must be Rh D-positive, because the father is homozygous Rh D- positive. The Rh D cells are foreign to the mother since she is Rh D-negative and so she mounted an antibody response against the antigen. The second child also must have been Rh D-positive and the mother´s antibody crossed the placenta, resulting in the disease. Atopic disease is a type I immediate hypersensitivity reaction that occurs in minutes following a second exposure to the offending allergen. The antigen could be ragweed, tree pollen, grass pollen, animal fur, or food products. Type IV hypersensitivity is delayed type hypersensitivity developing in response to viral fungal, and intracellular bacterial antigens. It involves CD4 helper lymphocytes, but not antibody. Immediate hypersensitivity is a type I reaction that occurs in minutes following a second exposure to the offending allergen. Common antigens include ragweed tree pollen grass pollen animal fur food products. Immune complex disease is a type III hypersensitivity reaction. It involves antibody and complement but the form of antigen antibody complement complexes. Rheumatoid arthritis, systemic lupus erythematosus, and serum sickness are examples of type III hypersensitivity.
-
Question 66 of 100
66. Question
1 pointsA donor liver arrives in New York from Chicago 7 hours after harvest. A biopsy is performed to check for ischemic injury. The sensitive oxidase P450 system is susceptible to ischemia. This system is located in which region of liver?
Correct
Incorrect
Explanation:
The liver can be divided into three zones. This division is based on the blood flow. Zone 1 is the closest to the blood supply known as the periportal zone; there is a zone 2, or intermediate (between 1 and 3); and zone 3 is called the pericentral vein zone due to its association with the terminal hepatic vein (historically thought to be in the center of the lobule). Zone 3 contains the P450 oxidase enzyme system, which is important in the chemical modification or degradation of chemicals, including drugs and endogenous compounds. Zone 3 is the most sensitive to injury because it is the farthest away from the blood supply. The P450 system is not located in the bile ducts. The intermediate zone, or zone 2, is the area that is the second most sensitive to ischemic injury. “Ito” cells are the fat-containing mesenchymal cells located in the space of Disse. This is the site of vitamin A storage. The periportal zone, or zone 1, is the area most sensitive to toxic injury. This is the area that shows infiltration with hepatitis.
-
Question 67 of 100
67. Question
1 pointsA 14 year old girl with primary amenorrhea has normal breast development but axillary and public hair is still fine and unpigmented. She is 5.5″ and weighs 105 lb. Pelvic exam shows a shortened vagina with no cervix or uterus. Serum LH is markedly increased. What is the likely cause of the amenorrhea?
Correct
Incorrect
Explanation:
This individual has an XY karyotype and testes (probably abdominal) but because of complete androgen resistance developed a female phenotype in utero. In the absence of androgen receptors, the indifferent external genital slit will differentiate into a vagina with clitoris and labia. However the vagina will end as a blind sac because Müllerian regression factor secreted by the testes will prevent the formation of a uterus or uterine tubes. The Wolffian ducts will also degenerate in the absence of androgen receptors. At puberty, the testes will respond to the increased LH by increasing testosterone secretion. Masculinization is not possible because of the absence of androgen receptors however significant gonadal or peripheral aromatization of testosterone to estrogen will produce breast enlargement and other female secondary sex characteristics. LH will remain high because of the absence of negative feedback by the testosterone. Pubic and axillary hair development which is also androgen driven, will not occur. Females with 17 alpha hydroxlase-deficiency are born with normal female internal reproductive tract and external genitalia. This is because the “default” program is for the female phenotype to develop in utero. With 17-alpha-hydroxdase deficiency sex steroids (estrogen in the case of females) cannot be synthesized and secreted. Affected females will not mature sexually at puberty, but will remain infantile. Since the patient described here exhibited breast development l7alpha-hydroxylase deficiency is unlikely. Furthermore, the absence of uterus or uterine tubes is not consistent with 17-alpha-hydroxylase deficiency. Constitutional delay in onset of menses may occur in certain families. It is thought to be due to slow maturation of the hypothalamic pituitary gonadal axis. However, growth velocity and development of breasts and pubic hair usually occur normally. A uterus and uterine tubes would be present since there is no disorder in embryologic development. Hyperprolactinemia can suppress the hypothalamic pituitary gonadal axis and produce amenorrhea, however it cannot explain the absence of pubic and axillary hair or the absence of a uterus and uterine tubes. Turner syndrome occurs in females with the XO genotype. They are typically short in stature, rarely reaching 5 feet. The germinal tissue in the ovaries is replaced with fibrous streaks. The internal reproductive tract is normal. Estrogen secretion is diminished in primary ovarian disorder, which leads to an increase in LH and FSH. The height of the present patient makes Turner syndrome unlikely. Furthermore, Turner syndrome could not explain the absence of a uterus and uterine tubes.
-
Question 68 of 100
68. Question
1 pointsA 55 year old male develops sustained, serve chest pain. He is diagnosed with MI and is admitted and stabilized, then released several days later. After three weeks he develops steady, burning chest pain. A friction rub is heard on auscultation. The probable underlying cause of this complication is
Correct
Incorrect
Explanation:
The patient has Dressler syndrome, which is thought to be an autoimmune phenomenon resulting in fibrinous pericarditis with fever and pleuropericardial chest pain several weeks (usually 3 weeks) after a myocardial infarction. Bacterial, fungal and viral infections can cause pericarditis, but would not be suspected in the setting of myocardial infarction. Chlamydia is not a usual cause of pericarditis.
-
Question 69 of 100
69. Question
1 pointsA 27 year old man with a past history of parathyroid surgery presented with galactorrhoea. His plasma FSH is 4.2 U/L (1-7), LH is 5.6 U/L (1-10), prolactin is 1654 mU/L (<360), TSH is (3.8 mU/L (0.4-5), IGF-1 is (33.4 nmol/L (7.5-37.3). The most likely diagnosis is which one of the following?
Correct
Incorrect
Explanation:
The story of galactorrhoea suggests hyperprolactinaemia and in the context of primary hyperparathyroidimsm suggests MEN type 1. MEN type 1 is an autosomal dominant condition and is associated with hyperparathyroidism, pancreatic neuroendocrine tumors and pituitary tumors.
-
Question 70 of 100
70. Question
1 pointsA 36-year-old woman presents with pains in the left arm. On examination she has wasting and weakness of the intrinsic muscles of the left hand, absent tendon reflexes in the left arm and impaired pinprick sensation in the left hand and forearm. What is the most likely diagnosis? Correct
Incorrect
Explanation:
Median and ulnar nerve lesions would not cause absent reflexes in the arm. Lower trunk brachial plexus (C8/T1) would not cause absent reflexes in the arm. Neuralgic amyotrophy affects the upper plexus (C5-6) and therefore does not cause wasting of small muscles of hand. Thoracic outlet syndrome will not cause absent reflexes. Syringomyelia typically causes loss of reflexes, spinothalamic sensory loss, and weakness. It can be asymmetrical initially.
-
Question 71 of 100
71. Question
1 pointsWhich of the following is a proto-oncogene?
Correct
Incorrect
Explanation:
Oncogenes are endogenous human deoxyribonucleic acid (DNA) sequences that arise from normal genes called proto-oncogenes. Proto-oncogenes are normally expressed in many cells, particularly during fetal development, and are thought to play an important regulatory role in cell growth and development. Alterations in the proto-oncogene can activate an oncogene, which produces unregulated gene activity, contributing directly to tumourogenesis. Oncogene alterations are important causes of:
- Rhabdomyosarcomas (ras oncogene)
- Burkitt´s lymphoma (C-myc is translocated intact from its normal position on chromosome 8 to chromosome 14)
- Neuroblastoma (N-myc proto-oncogene is seen in a proportion of patients with poor prognosis).
They should be contrasted with tumour suppressor genes. In this situation, the genes normally down regulate cell growth, and require inactivation to allow malignant growth. Examples include retinoblastoma.
-
Question 72 of 100
72. Question
1 pointsWhat is true regarding cystic fibrosis?
Correct
Incorrect
Explanation:
The prevalence of the cystic fibrosis gene at around 1 in 20 of the population would suggest that this provides some evolutionary survival advantage. Like sickle cell trait offering a survival advantage to malaria, it is believed that the cystic fibrosis gene offers a possible survival advantage against cholera and enteropathogenic bacteria. Cholera opens chloride channels, letting chloride and water leave cells. The CFTR protein does just the opposite, closing chloride channels and trapping salt and water in cells, which dries out mucus and other secretions. A person with CF cannot contract cholera, because the toxin cannot open the chloride channels in the small intestine. Carriers of CF enjoy the mixed blessing of a balanced polymorphism. They do not have enough abnormal chloride channels to cause the labored breathing and clogged pancreas of cystic fibrosis, but they do have enough of a defect to prevent the cholera from taking hold. During the devastating cholera epidemics that have peppered history, individuals carrying mutant CF alleles had a selective advantage, and they disproportionately transmitted those alleles to future generations. However, because CF arose in Western Europe and cholera in Africa, perhaps an initial increase in CF herterozygosity was a response to a different diarrheal infection.” (Read more about balanced polymorphism).
-
Question 73 of 100
73. Question
1 pointsWhich of the following concerning Corynebacterium diphtheriae is correct?
Correct
Incorrect
Explanation:
Corynebacterium diphtheriae is a gram positive, non-spore forming, pleomorphic bacteria that is also a facultative anaerobe. Corynebacterium diphtheriae causes diphtheria.
Typically diphtheria attacks the respiratory system, but may also affect the skin, conjunctiva and external genitalia. Signs and symptoms include sore throat, fever, and swelling of lymph nodes in the neck and general malaise. As the disease progresses diphtheria toxin is secreted. This destroys the membrane surface of the affected areas and replaces them with a greyish tough leathery “pseudomembrane” made of dead tissue, leukocytes and bacteria.
Toxin could also affect the heart, nerves and other organs in the body causing heart failure, nerve damage or suffocation.
Toxin can be neutralised by the immune serum produced by the host cells. Diphtheria is transmitted from person to person. Human beings are the main reservoir. -
Question 74 of 100
74. Question
1 pointsOsteomalacia may be expected in:
Correct
Incorrect
Explanation:
Osteomalacias may occur with vitamin D deficiency.
Mercury poisoning or any heavy metal poisoning causes an acquired Fanconi syndrome with distal renal tubular acidosis. -
Question 75 of 100
75. Question
1 pointsWhich of the following is NOT true of the genetics of cystic fibrosis?
Correct
Incorrect
Explanation:
The large number of mutations in the cystic fibrosis gene, which have been identified, make DNA diagnostic technologies unfeasible. Cystic fibrosis is an autosomal disorder, resulting from mutations in the gene located on chromosome 7. Its prevalence varies with the ethnic origin of a population, being most common in North American and European Caucasians. The most common mutation is a 3-base pair deletion which produces an absence of phenylalanine at the amino acid position 508 (B). This amino acid mutation at position 508 causes improper processing and intracellular degradation of the cystic fibrosis transmembrane regulator (CPT R) protein.
-
Question 76 of 100
76. Question
1 pointsPseudomyxoma peritonei is caused by which of the following?
Correct
Incorrect
Explanation:
Pseudomyxoma peritonei is a disease caused by a mucinous cystadenocarcinoma of the ovary. It is characterized by abdominal distension and pain, and occurs because the tumor cells secrete a large amount of mucus. The other diseases listed do not cause this condition.
-
Question 77 of 100
77. Question
1 pointsA 45 year old farmer presents presents with cough, shortness of breath, fever, and myalgias. He is diagnosed as having pneumonitis. The most likely etiology is
Correct
Incorrect
Explanation:
This patient has silo filler´s disease, which results in lung injury secondary to acute chemical pneumonitis. The offending agent is nitrogen dioxide, which is released from decomposed corn silage. The initial exposure can result in tracheobronchitis, but further exposure can lead to progressive pulmonary insufficiency. Irritating gases such as chlorine can cause an acute chemical pneumonitis in patients exposed in chemical and plastic industries and people who disinfect water with chlorine. Sulfur dioxide is a chemical gas which causes irritation of the lungs and pneumonitis in patients exposed while working in the paper industry or the smelting industry processing sulfide containing ores. Phosgene is used in the production of aniline dyes and also results in toxicity of the lungs from inhaled gases. Toluene di-isocyanate, liberated during the manufacture of polyurethane foams, causes an asthma-like reaction with wheezing, coughing, and chest pain.
-
Question 78 of 100
78. Question
1 pointsA 44 year old man is diagnosed with Vincent´s angina after physical examination reveals pharyngeal ulcers immersed in purulent exudate. The most likely infectious agent responsible for this is which one of the following?
Correct
Incorrect
Explanation:
Vincent´s angina, a necrotizing pharyngitis, is caused either by Staphylococcus aureus or Streptococcus pyogenes. In contrast, Borrelia burgdorferi causes Lyme disease, Coxiella burnetti causes Q fever, Rickettsia ricketsii causes Rocky Mountain spotted fever, and Francisella tularensis causes tularemia.
-
Question 79 of 100
79. Question
1 pointsCandida albicans is part of the normal human flora. Which of the following is FALSE regarding disease caused by Candida albicans?
Correct
Incorrect
Explanation:
An intact integumentary system prevents invasion of normally colonizing organisms. Disruption of these normal barriers by catheters, endotracheal tubes, severe burns, or abdominal surgery predisposes to deep invasive or disseminated disease. Polymorphonuclear leukocytes and monocytes are the primary defense against Candida albicans. Intracellular killing is primarily dependent on the myeloperoxidase, hydrogen peroxide, and superoxide anion systems. Decreased numbers of white blood cells are seen in severely immunocompromised persons. Thus, the inflammatory response may be minimal or even absent, despite severe disease. Dysfunction of T-cells predisposes to mucocutaneous disease. In cutaneous candidiasis, the characteristic picture is chronic dermatitis with yeasts confined to the stratum corneum. Microabscesses, interspersed in normal tissue, are a characteristic pathologic finding in visceral candidiasis.
-
Question 80 of 100
80. Question
1 pointsWhich of the following statement is FALSE regarding pyrogens?
Correct
Incorrect
Explanation:
Exogenous pyrogens generally act primarily by inducing formation of endogenous pyrogens. This occurs by stimulation of the host´s cells, primarily macrophages, but also monocytes and lymphocytes. The majority of exogenous pyrogens are microorganisms, including their toxins. The best studied of these is endotoxin, also known as lipopolysaccharide or LPS, which is found in the outer membrane of all Gram-negative bacteria. Endogenous pyrogens are polypeptides, which after gaining entrance to the circulation, produce fever at the level of the hypothalamic thermoregulatory center. Regulatory polypeptides produced by nucleated cells are known as cytokines. They mediate fever as well as mediate an acute phase response. No single cytokine provokes a febrile response of the magnitude equivalent to that produced by endotoxin.
-
Question 81 of 100
81. Question
1 pointsVascular abnormality that may result in bleeding due to immune complex mediated vasculitis is
Correct
Incorrect
Explanation:
Henoch Schonlein purpura is a vasculitis disorder associated with immune complexes.
-
Question 82 of 100
82. Question
1 pointsAlleles that are considered most closely linked with type I diabetes include
Correct
Incorrect
Explanation:
The HLA locus DR3 is most highly linked with type I diabetes mellitus. The other four loci are not
-
Question 83 of 100
83. Question
1 pointsWhat autoantibodies are found at the highest frequency in Wegener´s granulomatosis?
Correct
Incorrect
Explanation:
Anti-neutrophil cytoplasmic antibodies are found in 95% of cases of Wegener´s granulomatosis. -
Question 84 of 100
84. Question
1 pointsThe organism that would be most likely to produce achalasia is which one of the following?
Correct
Incorrect
Explanation:
Trypanosoma cruzi is a major secondary cause of achalasia. The others are not routinely associated with causing achalasia.
-
Question 85 of 100
85. Question
1 pointsKernicterus is developed by deposition which compoung in the basal ganglia?
Correct
Incorrect
Explanation:
Accumulation of unconjugated bilirubin in the basal ganglia is responsible for this syndrome of bilirubin encephalopathy. Iron, conjugated bilirubin, biliverdin, and galactosamine are not involved in kernicterus.
-
Question 86 of 100
86. Question
1 pointsWhich one of the following describes a hydatidiform mole?
Correct
Incorrect
Explanation:
A hydatidiform mole is a benign neoplasm of the chorion. It is not a malformed fetus, a chorionic cancer, a type of uterine cancer, or a cause of peritoneal carcinomatosis, or Meigs´ syndrome, which is usually the result of ovarian carcinoma.
-
Question 87 of 100
87. Question
1 pointsA 22 year old male presents to the ER with sudden onset of high fever, severe dyspnea, and malaise. His history is significant for IV drug abuse. A CD4 cell count of 73/microliter is found. Bronchoalveolar lavage reveals cyst like organism with cup shape with appearance like “crushed tennis ball”. The most likely pathogen is
Correct
Incorrect
Explanation:
Patients with AIDS are prone to a variety of opportunistic infections, and Pneumocystis jiroveci pneumonia represents one of the most common initial presentations. This organism is a ubiquitous atypical fungus that may cause an acute, life threatening acute interstitial pneumonia in immunocompromised patients. P. jiroveci appears as a 5 µm cystlike organism with a characteristic cup shape that has been likened to a “crushed tennis ball.” Silver staining allows its identification in bronchoalveolar lavage. A frothy exudate filling the alveolar cavities is a characteristic finding in patients with Pneumocystis pneumonia. Pneumocystis has been classified as a fungus by ribosomal RNA analysis, but it is an unusual member of the group. A gram positive catalase positive coccus describes the genus Staphylococcus. Staphylococcus does not silver stain and is not a common cause of dyspnea in AIDS patients. An encapsulated monomorphic fungus describes the genus Cryptococcus. This genus causes dangerous encephalitis in the immunocompromised person, but is not a common cause of pneumonia in AIDS. An intracellular parasitic protozoan describes Toxoplasma gondii. Toxoplasma may cause encephalitis in AIDS patients, but it would not be expected to cause pneumonia. Intracellular yeast describes Histoplasma capsulatum. This agent causes acute and chronic pulmonary disease in AIDS patients.
-
Question 88 of 100
88. Question
1 pointsA 46 year old Boy Scout notices flu like symptoms two weeks following a camping expedition. He presents after a low grade fever, myalgia, and malaise have been present for several days. Examination reveals an erythematous rash on one calf that consists of concentric rings. The agent that is transmitted by the same vector as the infectious agent in this condition is which one of the following?
Correct
Incorrect
Explanation:
This man has the symptoms of Lyme disease, caused by the large spirochete, Borrelia burgdorferi. The vector of Lyme disease is the Ixodes tick, which is also the vector which transmits Babesia microti, a sporozoan parasite related to Plasmodium, and Ehrlichia, which is an organism related to the Rickettsiae. The rickettsiae are prokaryotes with a cell wall, so they are not represented on the list.
A eukaryote with a cell wall describes a fungus; yet there are no fungi transmitted by the Ixodes tick. An infectious protein is a prion. These are the agents of so called slow viral diseases; and they are not known to be arthropod transmitted.
A prokaryote without a cell wall would describe the family Mycoplasmataceae, which include the genera Mycoplasma and Ureaplasma. A prokaryote without muramic acid in its cell wall describes the Chlamydiaceae, which include the genera Chlamydia and Chlamydophila -
Question 89 of 100
89. Question
1 pointsMagnetic resonance image of a 46 year old woman shows marked degeneration of the caudate nucleus. Based on her neuroimaging studies, which would be the most likely symptoms observed in this patient?
Correct
Incorrect
Explanation:
This patient most likely has Huntington disease. The caudate degenerates markedly in this disease although the putamen and nucleus accumbens also degenerate. Atrophy of the caudate and putamen can make the lateral ventricles appear very large when imaged. Bradykinesia, resting tremor, and difficulty initiating movements are typical of Parkinson disease, which results from midbrain substantia nigra degeneration. Intention tremor and akinesia are signs of cerebellar dysfunction. Moon facies, amenorrhea, and hypertension are typical of Cushing syndrome. Additional symptoms include truncal obesity, increased facial hair, acne, irritability, depression, and decreased energy. This can be caused by an ACTH-secreting pituitary adenoma. Motor and vocal tics are indicative of Tourette syndrome.
-
Question 90 of 100
90. Question
1 pointsAn elderly critically ill male patient undergoes abdominal CT scan. Bilateral adrenal enlargement due to hemorrhage is found. Adrenocortical assessment reveals primary adrenal insufficiency. What would further lab testing reveal?
Correct
Incorrect
Explanation:
Primary adrenal insufficiency occurs when the adrenal glands fail to produce adequate cortisol. Cortisol secretion is normally stimulated by ACTH release from the anterior pituitary. Once cortisol is released, it feeds back to the pituitary to inhibit further ACTH secretion. Therefore, if cortisol secretion was impaired, ACTH levels would be elevated rather than reduced.
Elevated cortisol does not occur in adrenal insufficiency. Instead, cortisol secretion would be decreased. Epinephrine is released by the adrenal medulla and could be reduced by adrenal insufficiency rather than increased by it. CRH is released by the hypothalamus and acts on the anterior pituitary to stimulate the secretion of ACTH from the anterior pituitary. It is inhibited when cortisol feeds back to turn off its production in the hypothalamus. Therefore CRH levels would be elevated in the absence of cortisol rather than decreased. -
Question 91 of 100
91. Question
5 pointsA 5 year old child presents because of accelerated growth. Lab data show that he has gigantism. The mother, a physiology graduate, asks if somatostatin could be used. The problem with using somatostatin is that it would also inhibit the release of which hormone?
Correct
Incorrect
Explanation:
Somatostatin is released by the hypothalamus, pancreas, and GI mucosa. Hypothalamic somatostatin acts chi the anterior pituitary to inhibit the release of thyroid stimulating hormone (TSH) and growth hormone (GH). Pancreatic somatostatin is released from the delta cells and acts to inhibit insulin, glucagon, and gastrin release. GI derived somatostatin is secreted in response to H and acts to inhibit the release of all GI hormones and gastric H secretion. While administering somatostatin would inhibit GH secretion in this patient, it could also result in hypothyroidism. This condition is usually treated by resection of the tumor, since there is a high chance of involvement of the adjacent brain structures.
Aldosterone release would not be altered by the administration of somatostatin. Aldosterone is primarily regulated by the rennin angiotensin system. In this system, renin is released from the juxtaglomerular apparatus in the kidney and acts by cleaving angiotensinogen into angiotensin I. Angiotensin I is converted into angiotensin II in the lungs by angiotensin converting enzyme. Angiotensin II stimulates aldosterone release from the adrenal cortex, and aldosterone acts on the distal regions of the nephron to increase Na reabsorption. ADH release would not be altered by somatostatin administration, as ADH is primarily controlled by plasma osmolarity. ADH is produced by the hypothalamus and stored in the posterior pituitary. When plasma osmolarity increases, hypothalamic chemoreceptors are stimulated and ADH is released. ADH acts on the renal tubule to increase water reabsorption in the distal regions of the nephron. Cortisol is not directly affected by somatostatin but is released in response to adrenocorticotropin (ACTH) from the anterior pituitary. Corticotropin releasing hormone (CRH) is released from the hypothalamus in response to stressful stimuli and stimulates the anterior pituitary to release ACTH. The subsequent release of cortisol acts to increase glucose and free fatty acids necessary to provide energy for overcoming the stressful situation. Testosterone is also not affected by somatostatin administration. Hypothalamic luteinizing hormone releasing hormone (LHRH) acts on the anterior pituitary to stimulate the release of luteinizing hormone (LH) and follicle stimulating hormone (FSH). LH acts on the Leydig cells of the testes to stimulate the synthesis of testosterone. Testosterone is released into circulation or diffuses into the Sertoli cells to facilitate spermatogenesis. -
Question 92 of 100
92. Question
1 pointsOn CT of a 44 year old male with headache and visual field defects a 5 cm mass is involving the optic chiasm and sella turcica. It is resected via a transsphenoidal approach. There is concern about possible anterior pituitary insufficiency. Which pairs of hormones are important to replace immediately?
Correct
Incorrect
Explanation:
The most dangerous hormonal deficiencies involve glucocorticoids and thyroid hormone. It has consequently become customary to “cover” patients for these potential deficiencies immediately after any event (including surgery and possible pituitary apoplexy) in which function of the anterior pituitary may be lost if it turns out that the function was not lost these hormones can be discontinued later. Deficiency of vasopressin produces decreased mineralocorticoid release and clinically recognizable diabetes insipidus (high urinary output with inability to concentrate urine), but replacement is usually not begun unless symptoms become apparent.
Gonadal steroids (not LH and FSH) are given eventually, but they are not urgently required. Growth hormone replacement is usually given only to children. Prolactin does not usually require replacement in cases of pituitary insufficiency. -
Question 93 of 100
93. Question
1 pointsA 71 year old woman dies 3 days after ICU admission. Clinical picture is characterized by fever, purulent sputum, and pulmonary infiltrates on CXR. Autopsy shows patchy areas of consolidation in both lungs, with neutrophilic exudate filling bronchi and bronchioles on microscopy. Gram staining reveals colonies of small gram negative bacilli. The most likely diagnosis is
Correct
Incorrect
Explanation:
Hospital acquired pneumonia (or nosocomial pneumonia) is defined as a pneumonia that mates more than 48 hours after admission to the hospital. It more commonly affects patients who are in the ICU or are mechanically ventilated. The most frequent causative microorganisms are Pseudomonas aeruginosa, Staphylococcus aureus, Klebsiella pneumoniae, E.coli, and Enterobacter. Pathologic features are those of acute bronchopneumonia, but the mortality rate is high, around 50%. Anaerobic pneumonia occurs in individuals predisposed to aspiration, i.e., those with depressed levels of consciousness, impaired deglutition, or tracheal/nasogastric tubes. Periodontal disease is an additional risk factor, as it leads to increased numbers of anaerobic bacteria in aspirated material. Necrotizing pneumonia, lung abscess, and pleural empyema are the most common pathologic lesions. Community acquired pneumonia is usually caused by pneumococcus (Streptococcus pneumoniae) and results in homogeneous consolidation of an entire lobe. Microscopically, a fibrinopurulent exudate fills the alveolar spaces. Lipid pneumonia is characterized histologically by large numbers of lipid-laden macrophages. This form of pneumonia occurs in association with obstructive bronchial lesions or aspiration of mineral oils. In primary atypical pneumonia there is a lympho monocytic infiltrate confined to interalveolar septa and interstitium. Mycoplasma pneumoniae is the most frequent etiologic agent.
-
Question 94 of 100
94. Question
1 pointsA 74 year old man with hepatitis C undergoes liver biopsy. While evaluating, the pathologist sees a lot of what he thinks is lipofuscin; but he is having difficulty differentiating this from hemosiderin. Histologic stain for which substance would be most helpful in making this difference?
Correct
Incorrect
Explanation:
Melanin, lipofuscin, and hemosiderin are all yellow brown, granular pigments that may be difficult to distinguish microscopically. The S 100 stain helps identify melanocytes (such as occur in malignant melanoma), and an iron stain helps identify hemosiderin. No stain specifically identifies lipofuscin, which is consequently a diagnosis based on subtle morphologic features and, if necessary, the absence of staining by S 100 or iron stains. Stains for calcium are used to identify bone tumors and dystrophic calcification. Stains for lipid are used to identify intracellular fat. Stains for mucin help distinguish between poorly differentiated adenocarcinomas and poorly differentiated squamous carcinomas. There are no available stains for sodium, which is a normal constituent of all tissues.
-
Question 95 of 100
95. Question
1 pointsA female presents because of fatigue. Labs reveal leukemia with a translocation from the long arm of chromosome 22 to chromosome 9. This translocation thing is associated with a favorable prognosis in her condition. The most likely diagnosis is
Correct
Incorrect
Explanation:
The presence of the Philadelphia chromosome, a translocation from the long arm of chromosome 22 to chromosome 9 [t(9;22)], is associated with a more favorable prognosis in patients with chronic myelogenous leukemia. Acute lymphoblastic leukemia (ALL) is the most common cause of leukemia in children. The presence of the Philadelphia chromosome is associated with a worse prognosis for the patient. This form of leukemia is also associated with a B-ALL translocation of the c-myc proto-oncogene of chromosome 8 to chromosome 14 [t(8;14)(g24;g32)]. Acute myelogenous leukemia (AML) is the most common acute leukemia in adults. The M2 subtype is associated with the t(8;21) translocation, and the M3 subtype is associated with the t(15;17) translocation. More than half the patients with chronic lymphocytic leukemia display one of several chromosomal abnormalities. This includes trisomy 12 (involves the h-ras proto-oncogene), translocation t(11;14) (involves k-ras and bcl-1 proto-oncogenes), and deletion (14q) or inversion (14q) (involves immunoglobulin heavy chain gene). Hairy cell leukemia is associated with the expression of Tartrate-resistant acid phosphatase (TRAP) on the surface of B cells.
-
Question 96 of 100
96. Question
1 pointsMarrow aspirates from an adult with Hemoglobin SS disease (sickle cell anemia) and another with normal Hemoglobin A are being studied. Marrow aspirate from sickle cell anemia patient could be identified because of its increased
Correct
Incorrect
Explanation:
Adults with sickle cell disease have undergone decades of accelerated RBC formation and destruction, leading to accelerated erythropoiesis in the bone marrow. Consequently, the bone marrow becomes hyperplastic, with marked increases in the number of normoblasts (erythroblasts) at the expense of marrow fat and marrow bone. Although the white cell and megakaryocyte lines are undiminished, there is a marked increase in RBC precursors and iron stores. Iron storage increases as a consequence of both chronic transfusions and increased dietary absorption; these increased iron stores can be appreciated with a Prussian blue stain.
-
Question 97 of 100
97. Question
1 pointsA 37 year old otherwise healthy obese male presents with lethargy. He takes no medication. He has a BMI of 36.4 kg/m2 and a blood pressure of 120/72 mmHg. Systemic exam is normal. Investigations reveal sodium of 141 mmol/L (137-144), potassium of 2.8 mmol/L (3.5-4.9), urea of 5.6 mmol/L (2.5-7.5), and Creatinine of 7.6 µmol/L (60-110). The most likely diagnosis is
Correct
Incorrect
Explanation:
Bartter´s syndrome is a mixed bag of disorders but most frequently characterised by an autosomal recessive condition consisting of juxta-glomerular cell hyperplasia and secondary hyperaldosteronism. A normal or low blood pressure is typical. It may present in childhood with weakness and failure to thrive but may present coincidentally in adulthood.
-
Question 98 of 100
98. Question
1 pointsWhich public health measure would reduce the incidence of iron deficiency anaemia?
Correct
Incorrect
Explanation:
The following would achieve primary prevention of iron deficiency anaemia:
- Provision of adequate iron supplements for premature and low birth weight infants in adequate dosage (2mg/kg of elemental iron per day)
- Not using unmodified doorstep milk in the first year of life. Although breast milk has a low iron concentration, the relative bioavailability is much higher than from modified or unmodified cow´s milk
- Not giving young children tea (this reduced iron´s bioavailability)
- Use of follow on or ordinary infant formulae in the second half of the first year of life
- weaning on to mixed feeding by 6 months of age
- Iron supplementation for all children in high risk groups.
-
Question 99 of 100
99. Question
1 pointsA 31 year old renal transplant recipient presented with non-Hodgkin´s lymphoma. The virus that is most likely to be of aetiological significance is which one of the following?
Correct
Incorrect
Explanation:
EBV associated lymphoproliferative disease may occur in individuals with inherited or acquired immunodeficiency syndromes. Approximately of renal transplant recipients develop post transplant lymphoproliferative disease (PTLD) in the first year following their transplant.
-
Question 100 of 100
100. Question
1 pointsWhich of the following abnormalities is associated with short stature?
Correct
Incorrect
Explanation:
Turner´s syndrome, 45 XO, is characteristically associated with short stature.
Klinefelter´s is associated with tall stature. In Fragile X, height is usually unaffected and homocystinuria may have a Marfan´s habitus.